LSAT and Law School Admissions Forum

Get expert LSAT preparation and law school admissions advice from PowerScore Test Preparation.

User avatar
 Dave Killoran
PowerScore Staff
  • PowerScore Staff
  • Posts: 5850
  • Joined: Mar 25, 2011
|
#42383
Complete Question Explanation
(The complete setup for this game can be found here: lsat/viewtopic.php?t=16021)

The correct answer choice is (B)

This is the first question to address the “honors” subgroup established in the initial conditions of the game. The conditions in the question stem establish the following 2-3-2 fixed numerical distribution setup:
J97_Game_#1_#6_diagram 1.png
Since table 1 can only have one sponsor from the honors subgroup, answer choice (A) can be eliminated. Conversely, answer choices (C), (D), and (E) can be eliminated since table 1 must have exactly one sponsor from the honors subgroup, and none of these answer choices meet that criterion. Alternately, answer choices (D) and (E) can be eliminated since they contain two sponsors from the P, Q, and Z group that must supply table 3 with two sponsors. By process of elimination, answer choice (B) is proven correct.
 Amar Pal
  • Posts: 2
  • Joined: Jul 11, 2019
|
#66423
Hello, I'm going back and reviewing some of the material from the Online course. This is the only question that I still can't seem to understand. I have a feeling that I am misinterpreting the question stem. Why can the correct answer not be A? I understand that answer choices C, D and E cannot occur or they would violate some rules of the game.

However, I don't quite comprehend how we reach the conclusion that table 1 can have ONLY ONE sponsor from the honor group. The stem just tells us that table 2 needs to have three sponsors and two of those must be honors. Does this not leave the possibility for the following set up:

Table 1: K, M
Table 2: L, V, Z
Table 3: P, Q

Assuming that what I wrote above is possible, therefore, answer choice A could be correct. :-?


Thank you for your time.

Cordially,
Amar Pal
 Malila Robinson
PowerScore Staff
  • PowerScore Staff
  • Posts: 296
  • Joined: Feb 01, 2018
|
#66435
Hi Amar,
The Honors sponsors are KLM, and according to the question you need 2 of those folks to be at Table 2. In your example you only have one of of them (L) at Table 2. To put that another way, for answer A if you put 2 of the 3 honors folks at Table 1 you would only have one left over to put at Table 2 and that violates the info that is presented in the question.
Hope that helps!
-Malila
 Amar Pal
  • Posts: 2
  • Joined: Jul 11, 2019
|
#66438
Malila Robinson wrote:Hi Amar,
The Honors sponsors are KLM, and according to the question you need 2 of those folks to be at Table 2. In your example you only have one of of them (L) at Table 2. To put that another way, for answer A if you put 2 of the 3 honors folks at Table 1 you would only have one left over to put at Table 2 and that violates the info that is presented in the question.
Hope that helps!
-Malila
Thanks, I completely forgot that V is not an honor when I was doing this problem. Because V always has to be grouped with L which is an honor, for some reason in my head I mistakenly believed V was an honor as well. I forgot the rules told us that only M, L and K are the honors. Nothing about V is stated just that it has to go with L.

Okay that explains it, thank you!
 litigationqueen
  • Posts: 12
  • Joined: Sep 23, 2020
|
#79923
Amar Pal wrote:
Malila Robinson wrote:Hi Amar,
The Honors sponsors are KLM, and according to the question you need 2 of those folks to be at Table 2. In your example you only have one of of them (L) at Table 2. To put that another way, for answer A if you put 2 of the 3 honors folks at Table 1 you would only have one left over to put at Table 2 and that violates the info that is presented in the question.
Hope that helps!
-Malila
Thanks, I completely forgot that V is not an honor when I was doing this problem. Because V always has to be grouped with L which is an honor, for some reason in my head I mistakenly believed V was an honor as well. I forgot the rules told us that only M, L and K are the honors. Nothing about V is stated just that it has to go with L.

Okay that explains it, thank you!
Hi,

I had the same issue with this question and also chose answer A for the same reasons.
However, I thought inferring that V was an honors recipient was appropriate given that L and V must go in the same group. How do I avoid making false inferences like this? How do I diffeentiate between making an inference based on the conditions given and making false inferences/asusmptions? Really confused about that and I don't want this handicap with Logic Games going forward.

Thank you,
Michelle
 litigationqueen
  • Posts: 12
  • Joined: Sep 23, 2020
|
#79945
litigationqueen wrote:
Amar Pal wrote:
Malila Robinson wrote:Hi Amar,
The Honors sponsors are KLM, and according to the question you need 2 of those folks to be at Table 2. In your example you only have one of of them (L) at Table 2. To put that another way, for answer A if you put 2 of the 3 honors folks at Table 1 you would only have one left over to put at Table 2 and that violates the info that is presented in the question.
Hope that helps!
-Malila
Thanks, I completely forgot that V is not an honor when I was doing this problem. Because V always has to be grouped with L which is an honor, for some reason in my head I mistakenly believed V was an honor as well. I forgot the rules told us that only M, L and K are the honors. Nothing about V is stated just that it has to go with L.

Okay that explains it, thank you!
Hi,

I had the same issue with this question and also chose answer A for the same reasons.
However, I thought inferring that V was an honors recipient was appropriate given that L and V must go in the same group. How do I avoid making false inferences like this? How do I diffeentiate between making an inference based on the conditions given and making false inferences/asusmptions? Really confused about that and I don't want this handicap with Logic Games going forward.

Thank you,
Michelle
Hello,

So upon re-reading the conditions of the game, I noticed that "only" is used in the question stem when describing who received honors. I think this is the type of language to be mindful of going forward as "only" is super restrictive. Therefore, V was never an honors recipient. Please advise.

Thanks,
Michelle
 Jeremy Press
PowerScore Staff
  • PowerScore Staff
  • Posts: 1000
  • Joined: Jun 12, 2017
|
#79970
Hi Michelle,

You're correct that the "only" in the scenario is the key here. In the scenario, we're told that "only K, L, and M will receive honors." Nothing in the subsequent rules, or the questions' conditions, will contradict or override that (unless you're given a "rule suspension" question at the very end of the game, in the last question).

Long story short, if you see an "only" in the scenario or the rules, you can't override it. And if you're tempted to read one of the rules as overriding it, resist that temptation and follow the language.

I hope this helps!

Get the most out of your LSAT Prep Plus subscription.

Analyze and track your performance with our Testing and Analytics Package.